Calculus: Early Transcendentals 8th Edition

Published by Cengage Learning
ISBN 10: 1285741552
ISBN 13: 978-1-28574-155-0

Chapter 4 - Section 4.4 - Indeterminate Forms and l''Hospital''s Rule - 4.4 Exercises - Page 313: 85

Answer

$$ \begin{aligned} L&=\lim _{x \rightarrow \infty}\left[x-x^{2} \ln \left(\frac{1+x}{x}\right)\right]\\ &=\frac{1}{2} \end{aligned} $$

Work Step by Step

$$ \begin{aligned} L&=\lim _{x \rightarrow \infty}\left[x-x^{2} \ln \left(\frac{1+x}{x}\right)\right]\\ &=\lim _{x \rightarrow \infty}\left[x-x^{2} \ln \left(\frac{1}{x}+1\right)\right]\\ & \,\ [ \text{Let} \,\ t= \frac{1}{x}, \text{so, as } x \rightarrow \infty, t \rightarrow \ 0^{+} . ]\\ &=\lim _{t \rightarrow 0^{+}}\left[\frac{1}{t}-\frac{1}{t^{2}} \ln (t+1)\right]\\ &=\lim _{t \rightarrow 0^{+}} \frac{t-\ln (t+1)}{t^{2}}\\ & \,\,\,\,\,[ \text{form is} \,\, 0 / 0\,\, \text {and by using L'Hôpital's rule we have} ]\\ &= \lim _{t \rightarrow 0^{+}} \frac{1-\frac{1}{t+1}}{2 t}\\ &=\lim _{t \rightarrow 0^{+}} \frac{t /(t+1)}{2 t}\\ &=\lim _{t \rightarrow 0^{+}} \frac{1}{2(t+1)}\\ &=\frac{1}{2} \end{aligned} $$
Update this answer!

You can help us out by revising, improving and updating this answer.

Update this answer

After you claim an answer you’ll have 24 hours to send in a draft. An editor will review the submission and either publish your submission or provide feedback.